Cash-back offer from April 23rd to 27th, 2024: Get a flat 10% cash-back credited to your account for a minimum transaction of $50.Post Your Questions Today!

Question DetailsNormal
$ 27.57

ALS\ACLS-RED CROSS FINAL EXAM WITH 100% CORRECT QUESTIONS AND ANSWERS

Question posted by
Online Tutor Profile
request

ALS\ACLS-RED CROSS FINAL EXAM WITH 100% CORRECT QUESTIONS AND ANSWERS

 

A patient is brought into the emergency department. The patient does not have a pulse. The cardiac monitor shows the following rhythm. The team interprets this as which condition? - answer-Ventricular tachycardia

A patient with acute renal failure experiences cardiac arrest. Just before the cardiac arrest, the patient's ECG showed peaked T waves. What might be causing the patient's cardiac arrest? - answer-Hyperkalemia

 

A member of the resuscitation team is preparing to defibrillate a patient in cardiac arrest using a biphasic defibrillator. The team member would set the energy dose according to the manufacturer's recommendations, which is usually: - answer-120 to 200 joules

 

A member of the resuscitation team is preparing to administer medications intravenously to a patient in cardiac arrest. The team member follows each medication administration with a bolus of fluid. How much would the team member give? - answer- 10 to 20 mL

The resuscitation team suspects that hyperkalemia is the cause of cardiac arrest in a patient brought to the emergency department. Which finding on a 12-lead ECG would confirm this suspicion? – answer - Wide-complex ventricular rhythm or tall, pointed T waves

 

A patient with dyspnea and a change in mental status arrives at the emergency department. The healthcare team completes the necessary assessments and begins to care for the patient, including initiating cardiac monitoring, pulse oximetry, supplemental oxygen and vascular access. The team reviews the patient's ECG rhythm strip, as shown in the following figure. Which agent would the team most likely administer? – answer  - Atropine 0.5 mg every 4 to 5 minutes

 

A patient experiencing an unstable bradyarrhythmia does not respond to atropine or transcutaneous pacing. Which intervention would the healthcare provider use next? - answer- Administration of an epinephrine infusion

 

A patient's ECG reveals a tachyarrhythmia. The patient is hemodynamically stable and has a heart rate ranging from 120 to 135 beats per minute. Based on the findings of the secondary assessment, which statement(s) by the patient would the team interpret as a possible contributing cause? - answer-1. "I've had a terrible cold with a horrible cough and fever the past week."

2.   "I've been so anxious lately because I just lost my job."

3.   "I've been vomiting for the past 2 days from a gastrointestinal bug."

 

A patient's ECG reveals a narrow QRS complex with a regular rhythm, indicating a narrow-complex supraventricular tachyarrhythmia. The patient is hemodynamically stable. Which intervention would be initiated first? - answer-Vagal maneuvers

 

A patient in the telemetry unit is stable. Cardiac monitoring indicates the patient has ventricular tachycardia with a pulse. Further assessment reveals that the corrected QT interval is greater than 0.46 seconds. Which treatment would be appropriate at this time? - answer-Synchronized cardioversion

 

An ECG strip of a patient in the emergency department reveals the following rhythm. Which feature would the healthcare provider interpret as indicating atrial fibrillation? - answer-Absence of discrete P waves and presence of irregularly irregular QRS complexes

 

 

 

A 30-year-old patient has been brought to the emergency department in full cardiac arrest. The cardiac monitor shows the following rhythm. Interpretation of this rhythm would suggest which of the following as a possible precipitating factor? - answer- Electrocution

Cardiac monitoring of a patient in cardiac arrest reveals ventricular fibrillation. What intervention would the team perform next? - answer-Administer 1 shock.

 

A patient has experienced return of spontaneous circulation (ROSC) after cardiac arrest. The healthcare team is conducting a secondary assessment to determine the possible cause of the patient's cardiac arrest. Before the arrest, the patient exhibited jugular venous distension, cyanosis, apnea and hyperresonance on percussion. The patient was also difficult to ventilate during the response. The team would most likely suspect which condition as the cause? - answer-Tension pneumothorax

 

A patient in cardiac arrest experiences return of spontaneous circulation. As part of

post-cardiac arrest care, the patient is receiving mechanical ventilation. Which finding(s) would indicate the need for change in the ventilator settings to optimize the patient's ventilation and oxygenation? - answer-1. SaO2 92%

2.   PaCO2 35 mmHg

3.   ETCO2 50 mmHg

 

After cardiac arrest and successful resuscitation, the patient has a return of spontaneous circulation. The patient is unable to follow verbal commands and has a Glasgow Coma Scale score of 7. Targeted temperature management is initiated. Which method(s) would be appropriate for the resuscitation team to use? - answer-1. Applying cooling blankets to the patient's body

2.   Giving an ice-cold IV fluid bolus

3.   Using an endovascular catheter

 

A 40-year-old patient in the waiting room of the primary care provider's office approaches a staff member and says, "I'm having really severe, crushing chest pain that is moving to both my arms." The patient is diaphoretic and dyspneic. Which action would be appropriate for the staff member to take? - answer-Activate the emergency medical services system.

Available Answer
$ 27.57

[Solved] ALS\ACLS-RED CROSS FINAL EXAM WITH 100% CORRECT QUESTIONS AND ANSWERS

  • This solution is not purchased yet.
  • Submitted On 02 Feb, 2024 04:08:09
Answer posted by
Online Tutor Profile
solution
ALS\ACLS-RED CROSS FINAL EXAM WITH 100% CORRECT QUESTIONS AND ANSWERS A patient is brought into the emergency department. The patient does not have a pulse. The cardiac monitor shows the following rhythm. The team interprets this as which condition? - answer-Ventricular tachycardia A patient with acute renal failure experiences cardiac arrest. Just before the cardiac arrest, the patient's ECG showed peaked T waves. What might be causing the patient's cardiac arrest? - answer-Hyperkalemia A member of the resuscitation team is preparing to defibrillate a patient in cardiac arrest using a biphasic defibrillator. The team member would set the energy dose according to the manufacturer's recommendations, which is usually: - answer-120 to 200 joules A member of the resuscitation team is preparing to administer medications intravenously to a patient in cardiac arrest. The team member follows each medication administration with a bolus of fluid. How much would the team member give? - answer- 10 to 20 mL The resuscitation team suspects that hyperkalemia is the cause of cardiac arrest in a patient brought to the emergency department. Which finding on a 12-lead ECG would confirm this suspicion? – answer - Wide-complex ventricular rhythm or tall, pointed T waves A patient with dyspnea and a change in mental status arrives ...
Buy now to view the complete solution
Other Similar Questions

No similar question exists

The benefits of buying study notes from CourseMerits

homeworkhelptime
Assurance Of Timely Delivery
We value your patience, and to ensure you always receive your homework help within the promised time, our dedicated team of tutors begins their work as soon as the request arrives.
tutoring
Best Price In The Market
All the services that are available on our page cost only a nominal amount of money. In fact, the prices are lower than the industry standards. You can always expect value for money from us.
tutorsupport
Uninterrupted 24/7 Support
Our customer support wing remains online 24x7 to provide you seamless assistance. Also, when you post a query or a request here, you can expect an immediate response from our side.
closebutton

$ 629.35